Key Takeaway Mistake
With conditionals, you can get the contrapositive. with some statements, you can reverse; with mo...
anadolorit on May 5, 2023
  • December 2010 LSAT
  • SEC5
  • Q9
1
Reply
Answer Choice A
It took me a second but I ruled out answer choice A because what it strengthens in the correlatio...
AndrewArabie on December 2, 2022
  • December 2010 LSAT
  • SEC5
  • Q22
4
Replies
Answer choice E
To me, answer choice E explains the difference between the results in the first study but I don't...
AndrewArabie on October 24, 2022
  • December 2010 LSAT
  • SEC5
  • Q17
2
Replies
Help!
Could you please explain why the other choices aren't correct? Thanks!
rmkrutz@crimson.ua.edu on February 5, 2022
  • December 2010 LSAT
  • SEC5
  • Q23
4
Replies
typos in this question make it difficult to see...
grammar problems
gmaramara on January 15, 2021
  • December 2010 LSAT
  • SEC5
  • Q13
1
Reply
LSAT Flex Prep Tests
Hi! I recently completed the LSAT Max course along with all of the diagnostic tests, and I am sta...
delaenajoy on August 24, 2020
  • December 2010 LSAT
  • SEC5
  • Q1
2
Replies
B and D
Can someone please explain why B and D are wrong
izzy on August 20, 2020
  • December 2010 LSAT
  • SEC5
  • Q13
2
Replies
Choice A
Why is A not correct?
Parker-Zopp on June 17, 2020
  • December 2010 LSAT
  • SEC5
  • Q22
2
Replies
Answer choice D
Hi Can you please explain why answer choice D is correct? I'm not seeing why E is incorrect? ...
erojas on May 21, 2020
  • December 2010 LSAT
  • SEC5
  • Q17
2
Replies
Why isn't B the correct answer?
I am confused because the argument in the question stem is reversing and that is exactly what B i...
izyat on May 12, 2020
  • December 2010 LSAT
  • SEC5
  • Q9
1
Reply
Hello Lsatmax!
I was wondering why would A be wrong ?
Masada on May 8, 2020
  • December 2010 LSAT
  • SEC5
  • Q10
2
Replies
How do we rule out (E)?
(B) and (E) seem to be pretty similar in what they are saying.. or it's just me who doesn't see t...
iameunkyoung@gmail.com on April 17, 2020
  • December 2010 LSAT
  • SEC5
  • Q11
3
Replies
Can you explain this?
Why not E?
nivensdc on March 14, 2020
  • December 2010 LSAT
  • SEC5
  • Q2
1
Reply
all answers
Please explain why the wrong answers are wrong. Thanks.
tomgbean on November 19, 2019
  • December 2010 LSAT
  • SEC5
  • Q25
1
Reply
C
Please explain why C is incorrect.
tomgbean on November 18, 2019
  • December 2010 LSAT
  • SEC5
  • Q16
1
Reply
D v B
Why is D incorrect? I was debating between D and B as the answer choice.
Meredith on October 20, 2019
  • December 2010 LSAT
  • SEC5
  • Q10
1
Reply
Choice E
What's wrong with choice E?
Meredith on October 20, 2019
  • December 2010 LSAT
  • SEC5
  • Q19
1
Reply
Why is E Correct?
I understand the explanation offered in the other discussion but I do not see what answer choice ...
jgoldman on September 25, 2019
  • December 2010 LSAT
  • SEC5
  • Q22
1
Reply
Difference between B and E?
Hello, I was struggling between picking B and E and eventually went with B. Can someone explain w...
Ame15 on September 23, 2019
  • December 2010 LSAT
  • SEC5
  • Q20
2
Replies
Can you please explain how to arrive at the cor...
Hello, I had some trouble with with this one. Can you please explain why the other answer choices...
dace on August 10, 2019
  • December 2010 LSAT
  • SEC5
  • Q14
1
Reply